Infinitely many solutions of a linear system of equations












0












$begingroup$



Find the value(s) of a real parameter k so that the system of equations has infinitely many solutions.



$x+y+z=1$
$2x+ky+3z=-2$
$3x+5y+kz=-1$




Please help, I don't know how to go about this. Would you have to try every combination of the equations until you get a contradiction?










share|cite|improve this question









$endgroup$












  • $begingroup$
    You dont want a contradiction, this gives you empty solution. You want to find a $k$ such that one of the equation can be obtained as a linear combination of the other two equation.
    $endgroup$
    – quantum
    Dec 11 '18 at 1:45










  • $begingroup$
    What conditions have you learned for a system of equations having an infinite number of solutions? Start with that.
    $endgroup$
    – amd
    Dec 11 '18 at 2:52
















0












$begingroup$



Find the value(s) of a real parameter k so that the system of equations has infinitely many solutions.



$x+y+z=1$
$2x+ky+3z=-2$
$3x+5y+kz=-1$




Please help, I don't know how to go about this. Would you have to try every combination of the equations until you get a contradiction?










share|cite|improve this question









$endgroup$












  • $begingroup$
    You dont want a contradiction, this gives you empty solution. You want to find a $k$ such that one of the equation can be obtained as a linear combination of the other two equation.
    $endgroup$
    – quantum
    Dec 11 '18 at 1:45










  • $begingroup$
    What conditions have you learned for a system of equations having an infinite number of solutions? Start with that.
    $endgroup$
    – amd
    Dec 11 '18 at 2:52














0












0








0





$begingroup$



Find the value(s) of a real parameter k so that the system of equations has infinitely many solutions.



$x+y+z=1$
$2x+ky+3z=-2$
$3x+5y+kz=-1$




Please help, I don't know how to go about this. Would you have to try every combination of the equations until you get a contradiction?










share|cite|improve this question









$endgroup$





Find the value(s) of a real parameter k so that the system of equations has infinitely many solutions.



$x+y+z=1$
$2x+ky+3z=-2$
$3x+5y+kz=-1$




Please help, I don't know how to go about this. Would you have to try every combination of the equations until you get a contradiction?







linear-algebra






share|cite|improve this question













share|cite|improve this question











share|cite|improve this question




share|cite|improve this question










asked Dec 11 '18 at 1:36









usernamesAreHardusernamesAreHard

434




434












  • $begingroup$
    You dont want a contradiction, this gives you empty solution. You want to find a $k$ such that one of the equation can be obtained as a linear combination of the other two equation.
    $endgroup$
    – quantum
    Dec 11 '18 at 1:45










  • $begingroup$
    What conditions have you learned for a system of equations having an infinite number of solutions? Start with that.
    $endgroup$
    – amd
    Dec 11 '18 at 2:52


















  • $begingroup$
    You dont want a contradiction, this gives you empty solution. You want to find a $k$ such that one of the equation can be obtained as a linear combination of the other two equation.
    $endgroup$
    – quantum
    Dec 11 '18 at 1:45










  • $begingroup$
    What conditions have you learned for a system of equations having an infinite number of solutions? Start with that.
    $endgroup$
    – amd
    Dec 11 '18 at 2:52
















$begingroup$
You dont want a contradiction, this gives you empty solution. You want to find a $k$ such that one of the equation can be obtained as a linear combination of the other two equation.
$endgroup$
– quantum
Dec 11 '18 at 1:45




$begingroup$
You dont want a contradiction, this gives you empty solution. You want to find a $k$ such that one of the equation can be obtained as a linear combination of the other two equation.
$endgroup$
– quantum
Dec 11 '18 at 1:45












$begingroup$
What conditions have you learned for a system of equations having an infinite number of solutions? Start with that.
$endgroup$
– amd
Dec 11 '18 at 2:52




$begingroup$
What conditions have you learned for a system of equations having an infinite number of solutions? Start with that.
$endgroup$
– amd
Dec 11 '18 at 2:52










2 Answers
2






active

oldest

votes


















2












$begingroup$

Add the first and second and subtract the third equation we have: $(k-4)(y-z) = 0$. Thus we require $k = 4$ to have infinite solutions.






share|cite|improve this answer









$endgroup$





















    1












    $begingroup$

    Firs you rewrite the system of equation as a matrix equation $Ax=b$. The system has a unique solution if and only if $det(A)neq0$. But since $det A=(k-4)(k-1)=0$. So the solution is unique if and only if $kneq 4$ and $kneq 1$.



    But if you check it further, when $k=1$ the system has no solution while when $k=4$ the system has infinitely many solutions. So $k=4$ is the only answer.






    share|cite|improve this answer









    $endgroup$













      Your Answer





      StackExchange.ifUsing("editor", function () {
      return StackExchange.using("mathjaxEditing", function () {
      StackExchange.MarkdownEditor.creationCallbacks.add(function (editor, postfix) {
      StackExchange.mathjaxEditing.prepareWmdForMathJax(editor, postfix, [["$", "$"], ["\\(","\\)"]]);
      });
      });
      }, "mathjax-editing");

      StackExchange.ready(function() {
      var channelOptions = {
      tags: "".split(" "),
      id: "69"
      };
      initTagRenderer("".split(" "), "".split(" "), channelOptions);

      StackExchange.using("externalEditor", function() {
      // Have to fire editor after snippets, if snippets enabled
      if (StackExchange.settings.snippets.snippetsEnabled) {
      StackExchange.using("snippets", function() {
      createEditor();
      });
      }
      else {
      createEditor();
      }
      });

      function createEditor() {
      StackExchange.prepareEditor({
      heartbeatType: 'answer',
      autoActivateHeartbeat: false,
      convertImagesToLinks: true,
      noModals: true,
      showLowRepImageUploadWarning: true,
      reputationToPostImages: 10,
      bindNavPrevention: true,
      postfix: "",
      imageUploader: {
      brandingHtml: "Powered by u003ca class="icon-imgur-white" href="https://imgur.com/"u003eu003c/au003e",
      contentPolicyHtml: "User contributions licensed under u003ca href="https://creativecommons.org/licenses/by-sa/3.0/"u003ecc by-sa 3.0 with attribution requiredu003c/au003e u003ca href="https://stackoverflow.com/legal/content-policy"u003e(content policy)u003c/au003e",
      allowUrls: true
      },
      noCode: true, onDemand: true,
      discardSelector: ".discard-answer"
      ,immediatelyShowMarkdownHelp:true
      });


      }
      });














      draft saved

      draft discarded


















      StackExchange.ready(
      function () {
      StackExchange.openid.initPostLogin('.new-post-login', 'https%3a%2f%2fmath.stackexchange.com%2fquestions%2f3034752%2finfinitely-many-solutions-of-a-linear-system-of-equations%23new-answer', 'question_page');
      }
      );

      Post as a guest















      Required, but never shown

























      2 Answers
      2






      active

      oldest

      votes








      2 Answers
      2






      active

      oldest

      votes









      active

      oldest

      votes






      active

      oldest

      votes









      2












      $begingroup$

      Add the first and second and subtract the third equation we have: $(k-4)(y-z) = 0$. Thus we require $k = 4$ to have infinite solutions.






      share|cite|improve this answer









      $endgroup$


















        2












        $begingroup$

        Add the first and second and subtract the third equation we have: $(k-4)(y-z) = 0$. Thus we require $k = 4$ to have infinite solutions.






        share|cite|improve this answer









        $endgroup$
















          2












          2








          2





          $begingroup$

          Add the first and second and subtract the third equation we have: $(k-4)(y-z) = 0$. Thus we require $k = 4$ to have infinite solutions.






          share|cite|improve this answer









          $endgroup$



          Add the first and second and subtract the third equation we have: $(k-4)(y-z) = 0$. Thus we require $k = 4$ to have infinite solutions.







          share|cite|improve this answer












          share|cite|improve this answer



          share|cite|improve this answer










          answered Dec 11 '18 at 1:40









          DeepSeaDeepSea

          71.3k54487




          71.3k54487























              1












              $begingroup$

              Firs you rewrite the system of equation as a matrix equation $Ax=b$. The system has a unique solution if and only if $det(A)neq0$. But since $det A=(k-4)(k-1)=0$. So the solution is unique if and only if $kneq 4$ and $kneq 1$.



              But if you check it further, when $k=1$ the system has no solution while when $k=4$ the system has infinitely many solutions. So $k=4$ is the only answer.






              share|cite|improve this answer









              $endgroup$


















                1












                $begingroup$

                Firs you rewrite the system of equation as a matrix equation $Ax=b$. The system has a unique solution if and only if $det(A)neq0$. But since $det A=(k-4)(k-1)=0$. So the solution is unique if and only if $kneq 4$ and $kneq 1$.



                But if you check it further, when $k=1$ the system has no solution while when $k=4$ the system has infinitely many solutions. So $k=4$ is the only answer.






                share|cite|improve this answer









                $endgroup$
















                  1












                  1








                  1





                  $begingroup$

                  Firs you rewrite the system of equation as a matrix equation $Ax=b$. The system has a unique solution if and only if $det(A)neq0$. But since $det A=(k-4)(k-1)=0$. So the solution is unique if and only if $kneq 4$ and $kneq 1$.



                  But if you check it further, when $k=1$ the system has no solution while when $k=4$ the system has infinitely many solutions. So $k=4$ is the only answer.






                  share|cite|improve this answer









                  $endgroup$



                  Firs you rewrite the system of equation as a matrix equation $Ax=b$. The system has a unique solution if and only if $det(A)neq0$. But since $det A=(k-4)(k-1)=0$. So the solution is unique if and only if $kneq 4$ and $kneq 1$.



                  But if you check it further, when $k=1$ the system has no solution while when $k=4$ the system has infinitely many solutions. So $k=4$ is the only answer.







                  share|cite|improve this answer












                  share|cite|improve this answer



                  share|cite|improve this answer










                  answered Dec 11 '18 at 1:56









                  user9077user9077

                  1,239612




                  1,239612






























                      draft saved

                      draft discarded




















































                      Thanks for contributing an answer to Mathematics Stack Exchange!


                      • Please be sure to answer the question. Provide details and share your research!

                      But avoid



                      • Asking for help, clarification, or responding to other answers.

                      • Making statements based on opinion; back them up with references or personal experience.


                      Use MathJax to format equations. MathJax reference.


                      To learn more, see our tips on writing great answers.




                      draft saved


                      draft discarded














                      StackExchange.ready(
                      function () {
                      StackExchange.openid.initPostLogin('.new-post-login', 'https%3a%2f%2fmath.stackexchange.com%2fquestions%2f3034752%2finfinitely-many-solutions-of-a-linear-system-of-equations%23new-answer', 'question_page');
                      }
                      );

                      Post as a guest















                      Required, but never shown





















































                      Required, but never shown














                      Required, but never shown












                      Required, but never shown







                      Required, but never shown

































                      Required, but never shown














                      Required, but never shown












                      Required, but never shown







                      Required, but never shown







                      Popular posts from this blog

                      Le Mesnil-Réaume

                      Ida-Boy-Ed-Garten

                      web3.py web3.isConnected() returns false always